The sphere at A is given a downward velocity v0 of

Chapter 13, Problem 13.39

(choose chapter or problem)

The sphere at A is given a downward velocity \(v_{0}\) of magnitude 5 m/s and swings in a vertical plane at the end of a rope of length l = 2 m attached to a support at O. Determine the angle u at which the rope will break, knowing that it can withstand a maximum tension equal to twice the weight of the sphere.

Unfortunately, we don't have that question answered yet. But you can get it answered in just 5 hours by Logging in or Becoming a subscriber.

Becoming a subscriber
Or look for another answer

×

Login

Login or Sign up for access to all of our study tools and educational content!

Forgot password?
Register Now

×

Register

Sign up for access to all content on our site!

Or login if you already have an account

×

Reset password

If you have an active account we’ll send you an e-mail for password recovery

Or login if you have your password back